LSAT and Law School Admissions Forum

Get expert LSAT preparation and law school admissions advice from PowerScore Test Preparation.

User avatar
 Dave Killoran
PowerScore Staff
  • PowerScore Staff
  • Posts: 5852
  • Joined: Mar 25, 2011
|
#45446
Complete Question Explanation
(The complete setup for this game can be found here: lsat/viewtopic.php?t=16890)

The correct answer choice is (A)

This question can be easily attacked by using the powerful inference that N must be second.

Answer choice (A): This is the correct answer choice.

Answer choice (B): This answer is incorrect because O cannot be second.

Answer choice (C): This answer is incorrect because J cannot be second, and because both L and N must be ahead of J.

Get the most out of your LSAT Prep Plus subscription.

Analyze and track your performance with our Testing and Analytics Package.